P = ρ GH is not a formula for calculating the internal pressure of a cylindrical liquid, but why can we use this formula for making the pressure changing body of a cylindrical solid

P = ρ GH is not a formula for calculating the internal pressure of a cylindrical liquid, but why can we use this formula for making the pressure changing body of a cylindrical solid


Because it is a regular column, f = solid density * V * G
P = f / s and because V / S = h, P = ρ GH
But if it's not regular (straight up and down), it can't be used
It is equivalent to a section of liquid column selected when exploring the internal pressure of liquid, and the principle is the same



Pressure formula
That is, the pulley block w placed vertically has =? W total =? N =?
Horizontal pulley block w has =? W total =? N =?
Pressure of liquid to vessel
Pressure of objects on tabletop
incorrect! Horizontal and vertical placement is not the same!


W = MGH weight, lifting height
Wtotal = FS pull and distance of pull
N = w has / W total
Horizontal placement is the same
Pressure of liquid to container P = P liquid GH liquid density G times height
The pressure of the object on the tabletop P = f / s pressure divided by the contact area
You should write the situation clearly. This general writing must be the same
It's just that sometimes the friction is multiplied by the distance, w = μ MGS
W is always equal to the pulling force F times the corresponding distance



The two identical containers of Party A and Party B are filled with water, and an iron ball and a wooden block are respectively placed in the two containers of Party A and Party B. if the pressure at the bottom of the two containers of Party A and Party B is p1.p2 at this time, what is the relationship between p1.p2?
To be specific, is there no difference between a wooden block and an iron ball?


The same height, the same liquid produces the same pressure
At this time, the two glasses of water are still full. The wooden block floats on the water, and the iron ball sinks at the bottom of the cup;
If the pressure of the liquid is the same, then the bottom of the cup is full of water
Of course, considering the contact point between the iron ball and the bottom of the cup, the pressure is still very large, because the buoyancy is far less than the gravity of the iron ball, and the iron ball still has a lot of pressure on the bottom of the cup, which produces a lot of pressure, but this is only the analysis of the contact point
This kind of problem generally refers to the pressure of the liquid at the bottom of the cup at this time



The theoretical basis for the equality of vertex angles is ()
fast


The theoretical basis for the equality of vertex angles is that the complementary angles of the same angle are equal
 
As shown in the picture,
Both ∵ 1 and ∵ 2 are complementary angles of ∵ 3,
The complementary angles of the same angle are equal
 



The proposition "equal to vertex angle" is ()
A. Definition of angle B. false proposition C. axiom D. theorem


It is a theorem that the vertex angles are equal



Are all diagonals equal
Please explain why


This statement is too absolute. If it is put in multiple choice questions, it will be wrong
Angles a and B are opposite vertex angles to each other, angles C and D are opposite vertex angles to each other, angles a and B are equal, angles C and D are equal, but angles a and C are not necessarily equal, because there is no relationship between them. But they are opposite vertex angles. Only opposite vertex angles are equal



How many pairs of buttresses are formed when two lines intersect in a plane? How many pairs of buttresses are formed when three lines intersect at a point?
In plane
How many pairs of buttress angles are formed when two lines intersect?
How many pairs of buttress angles are formed when three straight lines intersect at one point?


In plane
Two lines intersect to form two pairs of opposite vertex angles which are less than the horizontal angle
Three straight lines intersect at a point to form six pairs of opposite vertex angles which are less than the horizontal angle



Observe the following figures to find the adjacent complementary angle (excluding the horizontal angle)
(1) As shown in Fig___ To apex angle;
(2) As shown in Fig. B, there are a total of___ To apex angle;
(3) As shown in Fig. C, there are a total of___ To apex angle;
(4) This paper studies the relationship between the number of straight lines and the logarithm of the opposite vertex angle in (1) ~ (3). If there are n straight lines intersecting at a point, it can be formed__ To apex angle;
In a hurry


(1)2
(2)6
(3)12
(4)n(n-1)



Observe the figure and answer the following questions. (1) in figure a, the common opposite vertex angle can be regarded as equal to ×; (2) in Figure B, the common opposite vertex angle can be regarded as equal to ×,
1) in figure a, there is a common opposite vertex angle, which can be regarded as equal to () × ()
(2) In Figure B, the common vertex angle can be regarded as equal to () × ()
(3) In Figure C, the common vertex angle can be regarded as equal to () ×; ()
(4) To explore the relationship between the number of straight lines and the logarithm of the opposite vertex angle in (1) - (3), if n straight lines intersect at a point, then () pairs of opposite vertex angles can be formed


There are two pairs of vertex angles when two lines intersect, six pairs of vertex angles when three lines intersect at a point, 12 pairs of vertex angles when four lines intersect at a point, 20 pairs of vertex angles when five lines intersect at a point, n pairs of vertex angles when n lines intersect at a point, n (n-1) pairs of vertex angles are less than horizontal angle



Can two horizontal angles be regarded as the right vertex angle?


It's not
The definition of vertex angle: after two lines intersect, there is only one common vertex, and the two sides of the two corners are opposite extension lines, so the two corners are called opposite vertex angles
Only one common vertex. Only. One
If it is a flat angle, the two straight lines will coincide. If they coincide, will there be only one intersection?